PDA

View Full Version : The answer to all our secrets


Argonautical
25th Sep 2006, 08:39
Went to Birmingham on Saturday PM and watched this Monarch A320 take off and noticed that the undercarriage was staying down for an unusually long time. Then heard the pilot inform the tower not to worry as they were leaving it down on purpose and off they went with it still down.

Anyone suggest a reason?

mesh
25th Sep 2006, 08:56
I have heard of this done before when the guys took off with high brake temps. If you pull the gear up then the temp falls much slower. I whon't comment on whether they should have taken off with high temps in the first place regarding perf and sop's. I have sat agast once though with my training Captain listening with interest when an F/O explained how they took off with 2 temps at 350 and just 'left the gear down to cool them a bit'. Find this hard to believe with Monarch though, very high standard of SOP's etc. They may have had an ecam indication, windshear warning etc wher they thought better just to climb up a bit before sorting things. When you raise the gear the doors open and in fact cause an increase in drag, if you have something happening windshear for instance then that's the last thing you want to do so you leave it be. One would have thought though they would have given some indication to ATC, but when you are climbing out of Birmingham and maybe you have a slight problem your RT could be effected.

HEATHROW DIRECTOR
25th Sep 2006, 14:47
Nothing too unusual and I've seen it lots of times. I told an Air France Airbus he had his gear down as he climbed away. "Affirm; we use it for take-off" was the response!

hobie
27th Sep 2006, 14:42
One would have thought though they would have given some indication to ATC,

I can rem some years ago a 747 Classic getting a call from the Tower after take-off, querying why it's U/C was left down? ..... just cooling the brakes came the reply ..... :)

Argonautical
27th Sep 2006, 14:58
Can I ask why the brakes would get hot on take off?

Max Angle
27th Sep 2006, 17:37
Most likely a brake problem (usally a tachometer) that means that the brake has been disconnected. The brakes are applied automatically during the retraction sequence to stop them rotating, if the brake is disconnected then you have to leave the gear down for 2 minutes after take-off to allow it to stop spinning. Quite why it's considered such a problem to have them rotating in the wheel well I don't know. Take-off and landing performance are both reduced via a table with one brake inop, and obviously if you have an engine failure you retract it anyway.

stanley
27th Sep 2006, 17:54
you leave the gear down when the brake is deactivated till the wheel stops (2 mins) as the wheel acts as a jyro and will put wear on the undercarriage because of the strain of moving the wheel through 90 degreases and slows the retraction on that side by up to 90secs (a300)

mesh
2nd Oct 2006, 10:40
when landing you apply manual braking and the temps go up. If for some reason (usualy a bit heavy footed on them) the brake temps are high you need to cool them. Some aircraft have brake cooling fans that you switch on to help. But if you havent got these and you are on a short turn around or the pilot applies to much braking in the taxi to takeoff then you have high temps again. Most likely that the pilot flying could have been under line training and a bit to heavy on the feet but as you have read there could be a thousand reasons

fatboy slim
2nd Oct 2006, 11:18
I was the FO. Captain called 'GO' just before 100kts when we got a BRAKES HOT Amber ECAM warning on one brake - only 300ish degrees. We decided to leave the gear down to cool it off and the Captain made a great call to the tower so everyone knew what was going on. Cooled it down, got them up, and off to wherever we were going. Probably a quick turnaround and the long taxi down to 15 put some heat into the brakes.

Argonautical
2nd Oct 2006, 12:28
Fatboy slim

I have it on video, if you want a small AVI of yourself let me know.

Kirk Biddlecombe
1st Nov 2006, 21:31
When Commercial style Aircraft steer during taxiing, what parts of the actual Aircraft are used to carry it out?
I always notice the actual nosewheel turning, but was wondering whether using left or right engine thrust and the rudder assists.
Also on a recent flight I was on (A320) I noticed the Aileron being raised on a wing (depending on the direction turned).

BOAC
1st Nov 2006, 21:49
Nosewheel normally via a steering wheel in the cockpit.

Use of aileron while turning on the ground is unusual - mind you, an Airbus............................:ugh:

fmgc
1st Nov 2006, 22:37
In A320 aileron movement will be when flight deck are doing the flight control checks. First PF then PNF (or Capt then FO depending on SOPs) does them so you will see them move twice.

Normally most of the steering is done by the nosewheel. Sometimes differential power can be used on tight turns but it can put more strain on the main landing gear.

mingalababya
2nd Nov 2006, 10:30
what about differential braking like in a light a/c? Is this done in jets when taxiing around tight corners?

GBALU53
2nd Nov 2006, 13:13
With the larger aircraft unless it is tight the nose wheel steering is anough for turning.

Where large aircraft need to turn on the runway to backtrack for a turn off yes the engine power does need to be used.

Light twins do not normally have sperate steering it is done through the nose wheel and braking and the engine help in the turn.

Aircraft from the commuter size seem to be the first aircraft to have a steering wheel or tiller (A Twin Otter has a tiller this is like a column change gear stick in a car up to turn right and down to turn left).

Sometimes controllers put pressure on pilots to do a tight 180 degree turn on the runway which does put a strain on the near undercarriage leg, an Aurigny Air Services ATR went into engineering with in the last week with a cracked undercarriage some tight turns may have caused or not helped this.

Our runway is 46 metres wide the turning circle i understand for a Boeing 757 is 45 metres so you have not got a lot of room either side, aircraft have gone farming before now by turning on the runway and going over the edge.

Not only the pilot being red faced but the controller could take some of the blame by better use of the taxiways or dumbell turning circles at the end of some runways thats my oppinion any way.

Mark 1
2nd Nov 2006, 13:47
Try a Tiger Moth for fun.

No brakes, no steering other than power and rudder or a handler on each wing-tip.

Downhill and downwind, you need to be very careful.

reely
3rd Nov 2006, 18:26
HELLO>>>FRIENDS¬!!!

What exactly are contrails?
What makes them occure?
Has ist anything to do with polution

hobie
3rd Nov 2006, 20:10
all you ever wanted to know about Contrails ....

try this Google ....

http://www.google.ie/search?hl=en&q=aircraft+contrails%3F&meta=

chiglet
3rd Nov 2006, 20:14
Contrails..... a "shorthand" for
Condensation Trails. The Hot Gases exiting a Jet Engine leave a "Vapour Trail" across the sky. It is merely the hot air [exhaust] meeting the cold air [atmosphere], and being visible. At different altitudes, the Con/Vapour trails are NOT visible due to atmospheric conditions...[temperature/pressure gradient..etc]. Converely, sometimes, due to the same "atmospheric conditions".....Con/vapour trails seem to last for eveeer ........:ok:
hope this helps
watp,iktch
thanx hobie

hobie
3rd Nov 2006, 20:28
thanx hobie

your very welcome .... :)

ishe
4th Nov 2006, 10:47
In the 747 There is one tiller per side for steering. Maximum deflection of the nose wheel is 70° either side of straight ahead. When 20° is reached, body gear steering is automatically engaged. With the rudder pedals, a maximum of 7° deflection can be reached.

It is a very strange feeling when looking out of the cockpit doing a tight turn, almost as though the aircraft is reversing or sliding sideways. (if that makes sense)

llanfairpg
6th Dec 2006, 00:51
Lots of rubbish above!

A320 carbon brakes noticably heat up virtually any time they are applied, if he was on a quick turnaround there may have been some residual heat left in them from previous landing and taxy in, although use of the brake fans should have cooled them. With a long taxy out you would need the fans on again.

There is a potential for dirt/crap and even debris to be thrown from the wheels on lift off which can go into the bay and rupture hydraulic lines or worse in the case where a tyre seperates so best to stop then spinning straight away, thats why they are braked, if you dont understand this say CONCORDE several times and you will!

There can be several reasons for leaving the gear down after take off, cooling has already been mentioned, or perhaps even a problem with the sequence, sometime if you think you have a problem that might be gear related and need a return you may not want to run the gear up in case you cant get it down again! Its not always a good idea to explain to ATC exactly what is going on because the reporter with the airband radio will write it up as an a dramatic emegency which isnt good for the companies reputation or image.

steviefartpants
6th Dec 2006, 06:17
A/C on a quick turnround,brakes still hot as no brake fans are fitted,A/C takes off brakes still hot after taxi(but below take off limit),cool them down with airflow before putting them into the hole.

Its as simple as that!

Carbon brakes are meant to get hot as they work better when upto working temp .

Gary Lager
6th Dec 2006, 13:32
Lots of rubbish above!

Certainly is isn't there, for instance:

Its not always a good idea to explain to ATC exactly what is going on because the reporter with the airband radio will write it up as an a dramatic emegency which isnt good for the companies reputation or image.

What are you, a professional pilot or the company's own version of Max Clifford? As pilots we are responsible for using all resources available to us to ensure the safety and security of our pax and crew, including talking to ATC about any non-standard/non-normal situations which may occur. You may even find them helpful from time to time.

FlightDetent
6th Dec 2006, 14:31
I understand that 330 ECAM logic does have some differencies over 32x, (not rated on 330 do not know which), but on 320 HOT BRAKES is inhibited during takeoff for good reasons. Only comes up at liftoff to advise crew not to raise the gear as opposed to woefully suggest stopping with a brake problem.

Fatboy, can you advise what is the BRAKES HOT inhibit phase for 330, pls?

FD.
(the un-real)

fmgc
6th Dec 2006, 14:49
Max Angle: Quite why it's considered such a problem to have them rotating in the wheel well I don't know.

The other reason that you might want the wheels to stop turning before you retract them is to avoid the stress on the landing gear caused by gyroscipic presession.

magpienja
18th Dec 2006, 17:58
Something I here at my local airport, any kind sole tell me what it means, something to do with engines starts me thinks.

Nick.

BOAC
18th Dec 2006, 18:17
Probably a 737, but may be other types too. When the APU is not able to suply air to start the engine, a ground-air unit is plugged in. On the 737 it plugs in on the right of the fuselage centreline. It is used to start No 1 (the left) and then disconnected (safely, away from the running engine - which is at idle only). Pushback takes place and number 1 is 'run up' to a medium power setting to provide enough air to start number 2 using 'crossbleed air'.

Other types may start number 2 on stand due to the location of the air connection.

A search will produce more. Here is one (http://www.pprune.org/forums/showthread.php?t=119909&highlight=crossbleed).

magpienja
18th Dec 2006, 21:27
Many thanks BOAC.

Nick.

P.S. just wondering why my post was moved.

BOAC
18th Dec 2006, 21:50
To the correct thread?:)
Have you read the announcement and this thread (http://www.pprune.org/forums/showthread.php?t=251117)?

sniper9652
19th Dec 2006, 09:53
Hi all, just been observing a BMI Baby 737 in the hold at 'MIRSI' on my SBS-1. The pattern it creates is an absolute perfect race track shape pattern! I take it that it is auto pilot and instruments that maintain this shape and pattern during the hold? any feedback from aircrew would be appreciated!

Regards, Steve

Wodrick
19th Dec 2006, 17:48
Post #21 A330 "HOT BRAKE" inhibit is from 80kts to lift off and again on touchdown to 80kts according to my FCOM which is not amended and quite old.

Tacitus
12th Jan 2007, 16:42
http://www.airliners.net/search/photo.search?id=0348313
This is quite an old photo but i was amazed by this . Boeing and Airbus models can do this ? And why a pilot would do something like this ?

Maz11
12th Jan 2007, 16:46
You can't do that on an Airbus. For the thrust reversers to deploy the ground squat switch needs to be depressed. In other words there needs to be weight on the wheels. You can go into the reverse position with the thrust levers but the thrust will remain at idle until the aircraft touches down.

i'm not sure about the Boeing but i would think they are the same.

Hope this helps. Maz

BOAC
12th Jan 2007, 16:54
Modern Boeing no, and I assume Airbus no also. There are 'switches' that need to be made before reversers can normally be deployed, such as weight on wheels, radio altitude and others. Maybe there was no protection on this Russian model.

Problems of doing it would be that if you could not retract the buckets and needed power...................................

I've no doubt some old Trident puke:D will be along soon to tell you that they used to use reverse on number 2 engine (yawn:) ). It may be that the B727 used it as well - in both cases to expedite descent.

Just beaten to the post by Maz:ok:

Tacitus
12th Jan 2007, 17:26
Thanks a lot for your replies . I've searched for western airplanes but i did not find a similar photo . I've never experianced something like this as a passenger but i assume it must be quite scaring . But i still don't understand why a pilot would do something like this instead of raising the speadbrakes for example

stanley
12th Jan 2007, 21:02
I seem to remember the DC8 had this reverse in flight for decent and short landing

Jet2
14th Jan 2007, 22:04
Thrust reversers on 737-3/4/500 will deploy below 10ft Rad Alt. Usually achieves a rather firm touchdown :eek:

TURIN
16th Jan 2007, 09:29
Lots of rubbish above!
There is a potential for dirt/crap and even debris to be thrown from the wheels on lift off which can go into the bay and rupture hydraulic lines or worse in the case where a tyre seperates so best to stop then spinning straight away, thats why they are braked, if you dont understand this say CONCORDE several times and you will!.
Yes there is isn't there?:rolleyes:
Most a/c I know only brake the wheels when gear selected up. They will spin away quite happily until hydraulic fluid is ported through the landing gear selector valve to the brakes.
As has been mentioned, the A320 MEL allows for a u/s brake, and SOPs state that the gear is left down for 2 mins after t/o to allow the wheel to spin down. (Gyroscopic forces and all that malarky apparently).:ok:

con-pilot
16th Jan 2007, 16:31
I've no doubt some old Trident puke will be along soon to tell you that they used to use reverse on number 2 engine (yawn ). It may be that the B727 used it as well - in both cases to expedite descent.

Yes you can reverse any or all engines on the 72 anytime one desires, as long of course you are at idle power. Now I do not believe that there were any procurers on reversing in flight published by Boeing, at least I never saw any. Personally I never reversed the number 2 in-flight to expedite a descent. I did reverse all 3 engines during a sim ride once when we wasting time at the end of a recurrency ride.

Instructor placed us over the outer marker of 26L at Lax, 250 Kts IAS, 10,000 feet, clean and said "Land it, straight ahead, no turns." I did, it was not easy and in no way would I ever try it in real life, unless I had no other option. Was kind of fun.

Groundloop
17th Jan 2007, 11:46
I've no doubt some old Trident puke:D will be along soon to tell you that they used to use reverse on number 2 engine (yawn:)
Not a Trident puke myself but remember many years ago at the hold at Glasgow as a BA Trident came in. Reversers deployed on outboard engines at about 30 feet.
Asked my instructor (who was a Trident FO) what that was about. Stated that it was quite common to "get the bl**dy thing on the ground!".
So, yawn away BOAC!

Digby-dude
23rd Jan 2007, 12:34
Hi All,

I noticed the BA465 from Madrid to LHR almost got canned on Sunday, the aircraft was a 75 (G-CPES I think) that seemed to have an issue when starting the r/h engine.

The engineering team arrived with a ladder and changed something that was situated inside the engine pylon (on the side of the aircraft body), a few mins later the engine was restarted and the aircraft departed.

Does anyone know the problem was? or what equipment may have been replaced within the pylons? Just to feed my intereset !! (And to learn more)

Thanks,

Adam

SteveSmith
19th Apr 2007, 16:40
I'm sat at gate 19 at NCL at the moment, waiting for a technical delay on my flight home to Bristol (EZ569). It's a 737 (not sure which type), registration ending JP.

As the crew went onto the plane, I noticed a bloke getting up into a cavity behind the nose gear a couple of times. It was obviously big enough for him to pull himself up into it completely, and sit with just his feet dangling out. Having done that a couple of times, he shut the cavity, and then the delay was announced.

What is in there, and might it give any clue as to why I'm still sitting here?:bored:

Thanks,

Steve.

forget
19th Apr 2007, 17:37
Electrics and Instruments bay......... most of the 'black boxes'. Here -

http://www.b737.org.uk/eandebay.htm

Now you can pop outside and tell him what's wrong.:)

spannersatcx
19th Apr 2007, 18:10
Electronic and Equipment (E&E) Bay

PK-GDU
23rd Apr 2007, 04:24
In May 1992, I occupied a left window seat of Cathay Pacific flight from HKG to LHR, departed 00.15. The CX flight flew the "great circle" which overflew, most of the hours ,ex USSR air space.

The F/A religiously told all passengers to keep the window shutters closed at all times. It was dark and everybody were sleeping, so it was okay with me, but a fellow passenger at the right side tried to have a peek, almost at once two of the female F/A urgently ,or in an almost "histeric" instruction requested him to close it immediately.

Fifteen minutes later, the cabin was in a total peace and silence. The light was dimmed to almost totally dark. I managed to lift my window shutter about an inch. Through the gap, in the faint moonlight, I could recognized two forms of jet fighters, down below at 7 o'clock position.

My question is, was it normal if a commercial flight overflying that area escorted or threaten by jet fighters? If yes then who paid the fuel bill for those fighters? Or it was just a close encounter like the unfortunate KAL 007 over Kamchatka ?

Thanks for any illuminating answer or information given to this question.

PK-GDU

clifftop
3rd Jun 2007, 13:02
Are any of you folks aware of such an incident?

An aircraft managing to land safely on a short runway (in an emergency for example), but then not having enough runway to take off, effectively permanently grounding the a/c??

forget
3rd Jun 2007, 13:11
Hardly likely. If it got in - it'll get out - stripped.

http://www.abpic.co.uk/photo/1001607

http://www.thirdamendment.com/wrongway.html

perkin
3rd Jun 2007, 13:47
A south African airline retired a 747-SP fairly recently which was donated to an aviation museum and landed on a runway at an aviation museum from where it would never take off again...quite a few videos of the landing if you google for it...the strip was almost too narrow for the 747's gear to fit on...!

Golf Charlie Charlie
3rd Jun 2007, 18:38
Concorde at Duxford ?

llondel
3rd Jun 2007, 19:49
Concorde would probably have managed to get out of Duxford OK if it wasn't for the fact that immediately after its arrival the runway got shortened so they could route the M11 through. However, I think the B52 at Duxford arrived after this time and it's the shortest runway ever used for one of them, with no chance of getting it up again.

HEATHROW DIRECTOR
5th Jun 2007, 11:54
VC-10 at Brooklands??

mutt
5th Jun 2007, 20:05
Mexican Gulfstream made an emergency landing in an Irish race course many years ago, they had to built a runway to get it out.

Saudi B747-100 landed with a full load of passengers at a military runway in Madras India, that runway was too short for the aircraft to takeoff, however it was stripped to make it capable of taking off from a 2nd runway.

Mutt

The_Steed
12th Jun 2007, 22:21
I came back from Bergamo to Edinburgh on Sunday with Jet2.com (733 - Ex BA?) and at V1 there was a strong vibration that felt like it was coming from the port engine.

Question is, is that normal or something to be concerned about?

Also, on both the outbound and return flights we were delayed by about an hour due to "air traffic flow-control restrictions" - is that a valid excuse or was the 25 minute turn around just too tight?

Thanks!

The Steed

jet2impress
12th Jun 2007, 22:30
So how do you know you were at V1? Flow control.... yes a valid excuse.

Can you describe the sound any better? Where were you sitting? Did the cabin crew look concerned?

The_Steed
12th Jun 2007, 22:36
We were rotating (apologies if that's not V1 :)).

I was in row 6 or 7C and it lasted for about two or three seconds - kinda hard to describe the noise other than like the noise when the engine first spools up accompanied with the shakes!

(Couldn't see the cabin crew but they went about their business as normal).

jet2impress
12th Jun 2007, 22:53
This may of been the nose wheel still spinning as the aircraft left the runway. It can pick up a bit of a shimmy sometimes as it leaves the tarmac. I can't think what else it could of been. If the cabin crew did not seem to notice anything, then don't worry. Also, the speed on rotation is referred to as Vr. Hope this helps.

NWT
14th Jun 2007, 11:59
With regards to steering the aircraft while taxiing. On a lot of the larger jets (i.e. 747), the nose wheels are steered using a hand operated steering 'tiller' (via cables and hydraulic actuators), there is also an input from the rudder pedals, if I remember correctly, only active below 80 knots and only moves the steering by about 6 degrees, mainly used for keeping plane straight during the take off and landing roll. Some crews will use during normal taxiing for small corrections. Also on the 747 while taxiing, once the nose wheels go past a certain angle (20 degrees ?) the main body gear also steers, but in the opposite direction, to relieve the stress on the legs and assist with turning the rear of the aircraft.

Impress to inflate
16th Jun 2007, 18:20
We elect to leave our gear down for longer than normal periods of time at times of heavy rain or snow/slush on the taxiway/runway to "blow" away water/slush/snow that may have stuck on the gear. Arriving at an offshore rig and not being able to drop the gear is very embarrassing.

Rainboe
21st Jun 2007, 21:10
'Relative Motion' dear boy. The fact that you, the viewer, is moving affects the apparent motion of another object. Have you noticed how lamp posts whizz past you in a car. Is the lamp post whizzing, or are you observing it to apparently? And who put that lamp post in a car anyway?

Now get back to your school work- it's still term time. To get anywhere, you have to be better than the next guy.

west lakes
22nd Jun 2007, 11:00
I saw a few planes that appear to be flying as if they're travelling in a diagonal line to their heading.


As Rainboe says relative motion but also can be an illusion. South af Ayr is a place known as Electric Brae, if you go there the illusion is basically it looks as though you are going uphill when in fact you are going down hill!
Also can be due to looking at fixed reference points. Saw a Tornado on Monday low flying in the distance. it looked as though it was in level flight at an extreme 30 - 40 degree nose up attiude, the second a/c of the pair appeared level. Realised it was my brain being tricked by the reference points is was comparing it to!

Mairi
25th Jun 2007, 10:55
i was on a KLM 737-300 flight from Schipol Amsterdam to Aberdeen today, and i was sitting in row 18f behind the wing and engines. I noticed once we took off and much turbulence a flap opened on the first pod on the wing behind the engine..... inside it looked like a cap was there... was it an inspection flap?? should it have been open??? it was above a slightly risen bit on the wing transparent looking?? was it the fuel pump access flap?? was it right to be open????

Rainboe
25th Jun 2007, 13:13
....a flap opened on the first pod on the wing behind the engine....
I'm sorry but your description makes no sense. you'll have to be more specific and accurate. Was it a speedbrake on top of the wing?

Johnny F@rt Pants
25th Jun 2007, 15:39
He's obviously just a pax who doesn't know the technicalities between flaps/spoiler/inspection panels. His description is accurate enough to be able to understand that what was seen was an inspection/access panel. What exactly it's access to is irrelevant in my opinion. As for whether it should have opened, the answer to that is certainly not, but it's not a problem these things do occasionaly open if they haven't been closed properly after regular maintenance. It just requires closing/replacing when you reach destination/maintenance base dependent on whether it's been damaged or not.
Hope that answer helps you out.

Doors to Automatic
25th Jun 2007, 15:46
Is this not a light on the 737?

FakePilot
25th Jun 2007, 16:46
One time I noticed a oval opening in the pylon above the engine. After landing they brought a ladder to that area but I left so it could have been anything. Curious, I watched next flight. There when we took off, but open after landing. I figured that was normal then.

What's up with that?

Mairi
25th Jun 2007, 17:28
Ok yes it must have been an inspection access point... To be clearer to the person who dosent understand...

I was sitting behing the wings, and you know how a wing has the white bits to help the flaps extend, the first one seems to be behind the engine and appears to lok like a fuel tank to me??? am i right guys???? above the hole was at the side of the pod and the pod leads to the engine under the wing..... if you follow it with your eyes..

I got very nervous at 24k feet and this flapping around and i take it was access to the fuel pump is that right???

i am a very technically minded 32 yr old woman and trying to learn!!! was annoyed it was open

BOAC
25th Jun 2007, 17:36
Don't worry, Mairi - he only flies the aircraft:). At least Johnny FP understood:ok:

Mairi
25th Jun 2007, 17:38
who the person who didnt know what im on about???? and was it wrong by the way

Rainboe
25th Jun 2007, 20:52
Right you lot settle down! I asked the lady perfectly nicely (for me) to explain more fully exactly where and what she was talking about.
Is this it? http://www.airliners.net/open.file/1170415/M/

No doubt an engineer will explain- I only count wings on my walkaround- 2 in number (hopefully). However it is not an important panel (the one on top of the fairing in front of the teardrop thingy?). It carries no load.

You might not believe it, but even the mighty 747, whose flap fairings (it just hides all the mechanism for holding and driving the flap out) are as large as bathtubs are broad, can fly without the fairing fitted. When they crack (they are only fibreglass), they are removed altogether from that position until a new spare can be delivered. It exposes all the mechanism to the air. Is it important? Not in the least. http://www.airliners.net/open.file/0595976/M/

So if the whole darn fairing is not needed, a little access panel in the fairing is less important than brushing your teeth at night to prevent decay if you wear dentures anyway.

Mairi
25th Jun 2007, 21:26
ok that now makes sense. I wish i could fly a 737 ort anything for that matter!!!

You been flying long??

no thats the flaps as you know see the pod like thing that support the flaps to move the hole was in the side of it

ok reading my threads i dont think i described it very well. I will try again.


Wings and flaps were normal

If you look at the wing from the window looking out you see the back of engine (if you behind it) and three white or silver pod like things attached to the wing i think they help the flaps extend....

The closest is in line with the engine at the back and looks a slightly different shape?? a fuel tank perhaps???

It had a compartment looking bit that blew open and the hole was in the side of this pod like thing and you could see inside it it looked like a cap or something????

its not the wing iteslf if you look at the pic its not the wing its the bit that sticks out and stays straight when the flaps go down like the pic is there but it was the one closest to the window behind the engine

if you look at the picture, see how the flaps have extended... the big solid straight bit that hasnt, one of them!!! now the hole that was open was at the side so you could have put your hand in and it would have ended up under the risen dome bit as you see in picture... is that the fuelpump??

do pilots inspect all the doors/panels/hatches etc on a plane before they fly or is it th engineers job

Big Frankie
25th Jun 2007, 21:31
I think Mairi is right. If a passenger can notice that. Why can't ground or flight crew?

If a really nervous passenger notices something like that. It could send them into a plind panic. Think on KLM

Rainboe
25th Jun 2007, 21:35
Mairi, I don't know what the risen teardrop domey thing is at the top, but on the side of this fairing, as far as I remember, you have channels, or tracks that a sticky outy thing on the ends and sides of the flaps run in. Inside the big fairing is a very large broom handle metal thing (screwjack) with a thick thread on it. This turns rapidly, and a large nut thing attached to the flap runs up and down it the screwjack. This makes the flaps move backwards and forwards, and the channel in the side of the fairing, into which the end of the flap thing runs, curves, making the flaps angle downwards according to how far they run along the screwjack. I don't recall a panel there because the flap would scrunch it. I think you saw the channel and it was meant to do that.

Now BOAC has been flying the beast for far longer than me, so I now hand over to him, he heap smart.............

Mairi
25th Jun 2007, 21:36
Im not blaming KLM big frankie but maybe we have a point here??? Perhaps i am too nervous??? I am used to paying incredible attn to detail and maybe too much, but i just would like to know whats potentially dangerous and whats not its nae fun at 24k ft!!! a hole does not look good!!! But perhaps its normal??

BOAC
25th Jun 2007, 21:40
Time to close the panel on this one, I feel.

No it should not have been open - most likely an engineer's error, but it could have been a failure of the fasteners.

It is very difficult to check all the panels on an aeroplane - counting wings is enough for some:)

It would almost certainly not cause a problem in flight, but panels coming open/falling on people on the ground - not good.

If you see this again, ask the cabin crew. They should be able to judge whether or not a pilot needs to come and have a look

I think we can close this one with Mairi a little wiser. If she wants to learn a little more, this excellent web site will help.

http://www.b737.org.uk/flightcontrols.htm

Big Frankie
25th Jun 2007, 21:41
Hi Mairi.

As usual. we'll get told what we want to hear from the big wigs. It's good to see others on this forum have got OUR intrests at heart.

I hope you weren't to distressed with the incident?

clifftop
26th Jun 2007, 01:41
About 10 years ago, I was sitting, as I still do now, watching the comings and goings at Liverpool. A 737 lined up on rwy 27, then throttled up and rolled to take off...

...as I watched it through binoculars, I saw a panel, about (?) 10+ feet behind the wing, on the underside was most definitely OPEN!!

I phoned the airport on my mobile to tell them what I'd seen, never heard anything from it??

Any ideas?:confused::confused::confused:

Flight Detent
26th Jun 2007, 04:17
Hi clifftop....

I've thought about your comments, and the only access doors in that sort of area are the potable water and toilet servicing doors, unless it was an AEW&C 737 in which case it may have been the aft EWSP system magazine(s), but then again, you may also have noticed the surfboard on the top!

Just kidding...FD.. :O

Capt Chambo
26th Jun 2007, 05:05
On the right hand side, below the tailplane? Most probably the pressurisation outflow valve.

North Stand Tier3
27th Jun 2007, 10:29
The 'sticky out' bits you refer to on the wing are the boat or canoe fairings.They provide an aerodynamic covering for the trailing edge flap drive system.The Number 3 and 6 fairings behind the engines are shaped differently from the others.The inboard flap section runs from the fuselage body fairing into these (The flap outboard of the engines is a totally different section).There is a channel cut out of the inboard face of the engine pylon boat fairing at production which allows the flap mechanism to run back along its track(which is hidden by the fairing).When the flaps are up this hole is filled by a spring loaded plate,referred to by Boeing as the 'Inboard Flap Midflap Torque Shaft Clearance Door'.
The CDL -an approved document that shows the crews and us engineers what panels/doors/components the a/c can fly without -allows the 737-300 to fly minus both these doors (one on each wing) with only a minimal enroute climb fuel penalty.Perfectly legal and perfectly safe.
We use this gap to access the flaps for part of their lubrication procedure but,trust me,it's not a panel you can manually open or leave 'undone'.The spring is very strong and the door is only opened and closed by extending/retracting the flaps.

Hope this helps!

Eboy
9th Jul 2007, 10:18
Well, I read the "big red letters at the top", so here goes:

How does a jet engine make a plane go?

According to the Wikipedia.org article on "jet engine", when the gasses emerging from the engine are faster than the speed of the aircraft, forward thrust is created.

I have heard a physicist say the above concept is incorrect. Instead, he says to focus on the combustion chamber of the engine. Fuel and air burn rapidly and that combustion pushes in all directions -- that part of the combustion in the forward direction pushes on the forward wall of the chamber, which is connected, ultimately, to the airframe, and pushes the plane forward. (The gasses emerging from rear of the chamber don't have as much to push against.)

Thank you for considering my question.

parabellum
9th Jul 2007, 10:34
Your physicist is correct.

Chesty Morgan
9th Jul 2007, 10:39
Your physicist is wrong.

Yes part of that expansion pushes on the forward part of the combustion chamber, but part of that expansion will also push on the rear part of the combustion chamber. Newton's Law, equal and opposite and all that.

A jet engine creates an area of high pressure at the rear/exhaust end, effecting a low pressure area at the front end. Everything moves from high to low pressure (like a wing) and voila! 'Tis a simple explanation as I haven't had any caffiene yet.

pilotmike
9th Jul 2007, 10:52
Your physicist is correct.

Or at least he would be if your 'jet' was powered by a rocket engine!

But this doesn't explain the operation of high bypass turbo-fan engines, where a large majority of the thrust is produced by the 'fan' part. This is essentially a shrouded propellor, and has nothing whatsoever to do with the 'jet' part, other than receiving its power from the (typically 1st stage) turbine arangement.

I'd go along with your original definition:

when the gasses emerging from the engine are faster than the speed of the aircraft, forward thrust is created.

When air is accelerated backwards (as in your description), thrust is generated. This applies equally to the fan part, and the jet part - both accelerate air backwards, and both generate forward thrust.

I think care is needed to distinguish between a 'jet' engine as used by the majority of airliners, and 'rocket' engines, which your physicist might well have been referring to.

PM

Control Column
13th Jul 2007, 21:52
Hi
Having spent many blissful lunch breaks watching ATR72’s pull onto stand and shut down I have always been curious as to why one propeller (I can’t remember if it’s the number 1 or 2 engine/turbine) slows down and then stops very abruptly, as if a brake has been applied, whilst the other propeller windmills to a leisurely stop. Does anybody know why? Does anybody care? I’m just curious.
My guess is, if it is number 1, that that is the side the pax disembark from and it is done for safety reasons, but how is it stopped?

Cheers

CC :)

h73kr
13th Jul 2007, 22:08
The No.2 propellor is indeed braked, by the ingeniously named 'prop. brake'. The reason is not for safety (if it was, the more likely candidate would be No.1 rather than No.2.

....and the reason is, for ATR an engine running with a braked prop. effectively becomes an APU. Quite clever really.

jerboy
13th Jul 2007, 23:15
alot of captains will only use this as a last resort where no GPU is available. if the prop brake failed in anyway anyone stood near it would quickly paint the surroundings red... not pretty.

Control Column
14th Jul 2007, 19:56
h73kr & jerboy

Thanks for that. I shall sleep better now!:)

Being an earthbound misfit due to hearing problems I hadn't realised the turbine was still running and just the propeller was braked. But I suppose if I had thought about it...:ugh:

Mod: Sorry if I put this in the wrong forum. I have seen & read the large red letters at the top and felt I fitted in to that category but was not sure if the question did.:O

'Good judgement comes from experience, and experience - well, that comes from poor judgement' - Anonymous:)

All the best.
CC

Merritt
17th Jul 2007, 07:36
As a PPLer I would always use full power on take-off but a recent discussion elsewhere suggests that this isn't always the case with commercial airliners & that 'thrust reduction' methods are employed.

Is this correct? If so, could someone explain what these measures are and why you would use them. Surely max takeoff power reduces the amount of runway required and gives the pilot more 'headroom' if an abort is required?

Cheers

Steve

BOAC
17th Jul 2007, 08:01
Please search for 'flex' and 'reduced' and you will find lots of information.

I'll give you a start.
http://www.pprune.org/forums/showthread.php?t=139948&highlight=flex

came from 'Questions'. Try 'Tech Log' also.

Don't forget to search properly.

Merritt
17th Jul 2007, 08:14
Thanks - that makes sense... Will search harder next time!

Cheers

Steve

D120A
17th Jul 2007, 11:32
I'd also go along with the Wikipedia explanation. Remember Isaac Newton and "Force = Mass x Acceleration"? Well, integrate that once with respect to time and you get "Force x Time = Mass x Change in Velocity" or, to use the accepted terms, "Impulse = Change in Momentum".

Thus, if you have a device, such as a pure jet engine or, even better, a big by-pass turbofan, that gives a huge mass-flow of air an increase in momentum (backwards), the force that did that (backwards) is going to have an equal and opposite reaction forwards - which is the thrust of the engine.

Measure the speed of the jet plume and a relatively simple calculation will indicate the thrust. Equally, if you measure the thrust (say on a test rig), you can calculate the speed of the jet plume.

wingboy
21st Jul 2007, 18:23
I don't know whether this question has been asked...but how powerful are jet engines? I remember seeing where it was said that the blast is like hurricane strength winds and can cause considerable damage. Are we looking at winds of more than 120mph? Could the blast uproot trees or damage houses?

barit1
21st Jul 2007, 19:26
Think of it this way: the engine on large widebodies can generate 60 - 70,000 lb. of thrust. If an object were to encounter the exhaust blast at close range, it would be feeling a large fraction of that 60,000 lb. force. It could easily overturn a large truck.

I know of a guy who drove his personal car about 100 yds. behind such an engine - it rolled him over completely, fortunately without injury, but the car was a mess. :ugh:

in-my-opinion
22nd Jul 2007, 22:37
Need a bit of education on this one! I live in (well under actually!) an area of what I am told is uncontrolled airspace. Over our village we have frequent visits from fast military jets (Tornados from RAF Leeming) , somewhat slower military trainers (Tucanos from RAF Linton -on-Ouse), some very high altitude commercial jets (oceanic traffic I assume) as well as an assortment of micro-lites, gliders, glider tow aircarft from a small local field and at night some turbo-props, that I assume to be cargo mail flights.

The first question is very simple - how does everybody avoid each other as it is uncontrolled..... bearing in mind the variety of speeds and sophistication of onboard equipment (do gliders have any????)

I appreciate that altitude has a bearing on this separation ...although the Tornados do cover a variety of altitudes, the others could probably all be segmented. Is this and any on-board radar, TCAS etc enough to deal with this?


My second question is that I am often intriqued by who is up there and going where - is there any website that you can access with your co-ordinates and see what is tracking thru the area?

thanks

HEATHROW DIRECTOR
23rd Jul 2007, 06:57
I've no doubt that someone will give you a really good explanation... but in the meantime, "Controlled Airspace" is a particular type of airspace with defined limits both horizontally and vertically within which an air traffic control service is provided and all aircraft are subject to ATC clearances. Uncontrolled airspace is much of the space outside CAS where pilots can largely fly without ATC clearance. However, even in those areas, ATC units provide a separation service to pilots who wish to participate by warning them of other traffic andaircraft flying in the immediate vicinity of aerodromes will be "contrtolled". Controlled Airspace does not always start at ground level and the high flying commercial jets you see will be in Class A Controlled Airspace and subject to ATC clearance with separation from all traffic.

I'm not aware of any web site where you can find live information about aircraft flying over you. A gadget called SBS "Virtual radar" is available to plug into your PC and this will display aircraft using a particular type of on board radar equipment called ADS-B. However, not all aircraft transmit the required information so will not be displayed. There are web sites where flights may be viewed on a map but they rely on the information I've just mentioned and it's usually delayed by 5-10 minutes for security reasons. Try www.openatc.com but don't expect to see any military jets on there!

Schmieglie
1st Aug 2007, 07:15
The reason why brakes are applied before the wheels are retracted is because a spinning disc (like a wheel) causes gyroscopic forces similar to that which keeps your bicycle upright. If the wheels are rotated for retraction, these forces can cause the undercarriage to shear off - not a pleasant thought.

One might think that these forces will not be large, but all those 747 wheels are quite large and heavy, causing substantial forces.

Flap Track 6
1st Aug 2007, 13:59
... how powerful are jet engines? I remember seeing where it was said that the blast is like hurricane strength winds and can cause considerable damage. Are we looking at winds of more than 120mph? Could the blast uproot trees or damage houses?

On a civil large bypass turbofan engine, the core flow exit velocity at full power will be circa 200 metres per second, which is circa 450 mph. If Barit1's friend was driving 100 yards behind a big fan engine, the blast he felt would have taken only half a second to travel that distance. So, yes, they are damned powerful. Obviously, bypass flow is significantly slower.

There are a number of 'training' videos around to demonstrate the power of these engines to airside workers. They usually take the form of some car or truck passing behind the aircraft and being blown away by the blast.

At Filton, they have a traffic light system on the A38 where it passes the end of the runway to stop traffic if there's something big sitting waiting to go for this very reason.

Mark 1
1st Aug 2007, 14:59
Flap Track 6,

The nozzle velocities are much higher than that.
Locally, they are just subsonic on most 70s and 80s large turbofans i.e. about 300m/s for the bypass and maybe 450m/s for the hot nozzle.

More recent engines have reduced the velocities a bit principally for noise reasons.

The expansion cone behind the jet expands at about a 7 degree angle, so the velocity reduces considerably with distance.

However, I do recall one test where a pitot rig was put behind an engine at full power to measure the blast about 60-100m away. Confused by the lack of manometer readings, we went outside to find the rig blown into the next field.

Certainly 100mph blasts may exist at some distance, so hold your hat on.

Rower
24th Aug 2007, 15:37
I know this may sound very basic but can anyone let me know how the radios are keyed on commercial a/c ? are the HF and VHF radios keyed from the same button and if so how are the varuos radios selected ?

BOAC
24th Aug 2007, 15:41
Same 'press to transmit' button/switch but selected via an audio selector panel. Have a look at http://www.b737.org.uk/ for 737 info.

airborneforever
30th Aug 2007, 14:30
happened in alice springs late last year me thinks.
BA flight to sydney,
one pax had a heart attack and plane landed in alice,
couldnt take off again with all pax and enough fuel.
long story short,
relief flight from yssy took requisite no. of pax and said 747 did a short field take off and barely made it
hope that helps,
A.F

wingboy
1st Sep 2007, 16:01
In the movie Air Force One, the 747 does some dramatic maneuvers when it lands on the airport. Can a jet do this for real? Also, it then takes off with what appears to be full flaps. Can a 747 or any other commercial plane take off with the flaps fully extended?

west lakes
1st Sep 2007, 17:53
Whilst not exactly answering your questions, this may give you a clue

http://www.pprune.org/forums/showthread.php?t=287911

JohnnyRocket
2nd Sep 2007, 14:22
I was travelling on a Jet Blue A320 on a calm evening last week when at around 18,000ft out of nowhere we hit severe turbulence for around 15 seconds. The aircraft vibrated, dipped slightly and lurched to the right. Then - all was calm.

A short time later, the captain came on to say that we had gone over the wake of a following aircraft.

This is only the second time I have experienced this - although this time is was at a much lower altitude.

I have heard how dangerous wakes can be - are they "less" dangerous if you are at a higher altidude?

Also, what is the means for escape? Do you simply keep flying on and hope you pass through it, or is there a proper procedure?

perkin
2nd Sep 2007, 14:57
I've had a similar experience climbing out of Heathrow when we crossed the wake of another aircraft, bit of a bump and a few sharp intakes of breath from around the cabin but no real drama. I think its only a major issue close to the ground, i.e. on take off and short finals, where the wake turbulence could potentially dump you straight into the ground...hence the spacing between a heavy and a following smaller a/c

west lakes
2nd Sep 2007, 15:36
The only advantage was many years ago on a flight to tennerife S, on approach had a member of cabin crew tidying a locker above my seat & stood on the edge of the seat, a bit of turbulance saw her sat on my knee - nice girl:D

merryterry
6th Sep 2007, 15:13
Probably get flamed on this one but I am aware that a regular question is asked about the PTU noise on a A320. The dog barking in the hold question.

Flown many flights in an A320 and never really noticed it. Last week flew back from Cork on Aer Lingus A320 and the bloody dog was barking all the way down to take off. Really frightened the wife who thought something was wrong with the plane. 'No problems' I said 'just a self-test on the Power Transfer Unit' as I had been taught to say on this forum.

Flew BHX-DUB-BHX yesterday on A320.Dog barked a couple of times on 2nd engine startup on both flights and then silent.

Was my experience with continual dog barking down to take off unusual?

perkin
6th Sep 2007, 15:22
I heard the barking dog sawing his way out of the hold last time I went on a 320...! ;)

clifftop
7th Sep 2007, 16:43
Can anyone tell me if it was normal for concorde to use less than all 4 afterburners in the early stages after take off.

I ask because a member of another (non aviation) website I use commented that he knew when concorde was late taking off because all four A/B's were lit. Now that sounds like cobblers to me, I always assumed they were all on OR all off. Cannot see what difference it's departure time would make in a decision to rocket skywards???

Cliff

Musket90
7th Sep 2007, 16:58
Never did see it take-off at Heathrow without all 4 afterburners on. Soon after take-off when at safe speed and height I think they were switched off for noise abatement.

clifftop
7th Sep 2007, 17:04
Musket 90 said:
"Never did see it take-off at Heathrow without all 4 afterburners on. Soon after take-off when at safe speed and height I think they were switched off for noise abatement."


That's what I thought. Thanks for the reply.

HEATHROW DIRECTOR
7th Sep 2007, 17:56
I must have cleared Concorde for take-off a thousand times or more at Heathrow and it always used all four.

clifftop
7th Sep 2007, 19:07
That'll do for me!:)

ChristiaanJ
8th Sep 2007, 10:28
For supersonic acceleration (Mach 1 to about Mach 1.7), Concorde needed all four reheats (too much asymmetric thrust with three).

So if for whatever reason one of the reheats didn't light at the start of the take-off roll (which occasionally did happen), the take-off was abandoned, and the aircraft would return to the stand.

Such a rejected take-off was a pretty straightforward affair, because the aircraft would usually not have reached more than about 50mph.

PPRuNe Pop
8th Sep 2007, 10:58
I am sure I was told by a Concorde Captain that No.4 was throttled back a tad on take-off. There was a good reason for it too.

clifftop
8th Sep 2007, 11:54
Any idea what the good reason was?:confused:

amanoffewwords
8th Sep 2007, 12:26
All explained at the bottom of this page (http://www.concordesst.com/powerplant.html) on the Concorde SST website (not that I understand a word of it of course :confused:)

clifftop
8th Sep 2007, 17:29
I think that clears that up then. A problem free take off would see all four afterburners at full whack after 60 kts, therefore all 4 flamethrowers should be clearly visible once airborne. Ipso Facto, the geezer to whom I originally referred is a muppet.

Cheers me dears!

Cliff

ChristiaanJ
8th Sep 2007, 19:53
Cliff,
I finally cottoned on to the weird reasoning your muppet must have been using.....
Is that some strange urban legend doing the rounds?

Nicholas49
11th Sep 2007, 19:23
I want to open a thread about something I've noticed on the last few short-haul flights I've taken. I'm not sure if this is an issue or not (I'm sure I'll soon be told if it isn't!).

I've noticed that some pilots on short-haul flights "rev" (sorry - don't know the technical term) the engines to check they're stable literally as they are still turning onto the runway and before they are lined up.

Is this a common and accepted practice? Do any professional pilots frown upon it? (For example, if it caused you to mess up your alignment.) Is it at all a sympton of a rushed take-off?

I'd appreciate your comments.

chiglet
11th Sep 2007, 20:11
I've been on the Flight Deck, and on approaching the Holding Point, have heard XXX123 are you ready Immediate?. If yes, then a/c is cleared for an "Immediate Take Off". A/c "spools up the engines" [advances the throttles] and enters the Runway...and departs. :ok:
IF not ready, then hold until they are.
Also in 35 years of [Civil] Aviation, I've lost count of the number of "rushed" departures :E
watp,iktch

Caudillo
11th Sep 2007, 20:14
Nothing to do with being rushed. It's a "rolling takeoff" - happens when all preflight checks are completed, the cabin is secure, and clearance to take off has already been given prior to entering the runway or whilst entering it. Of course you check with the guy in the seat next to you that he is happy to continue and off you go. Useful to keep the flow up on a busy strip.

BYALPHAINDIA
11th Sep 2007, 21:40
I Think we have seen more 'rushed' T/Offs since the low cost has boomed, I can't remember the last time I saw an Aircraft doing an 'on the brakes roll off'

I still miss the DC9'S at the end of LBA'S R14, Doing a full thrust roll off the brakes!!:D

I remember an FR 737-200 in a rush going off R14 years ago, He was in a rush and had to be told to wait!!

The tower said wait, And he replied I can't wait!!:ugh:

Bullethead
12th Sep 2007, 01:11
G'day Nicholas,

The reason sometimes that jet aircraft are stopped on the runway and part power is applied before takeoff thrust is set is because sometimes the engines initially accelerate unevenly from idle. So rather than setting takeoff thrust straight from idle and getting a large thrust imbalance, part power is set, the engines stabilised and then takeoff thrust is set.

When doing a rolling takeoff you can advance the throttles as you enter the runway and if it all looks good then you can set takeoff thrust without getting a thrust imbalance.

It's not a rushed takeoff but an expedient way of entering the runway and taking off without stopping.

I'd be surprised if any jet operators set takeoff thrust on the brakes.

Regards,
BH.

Hey Mods, Has this thread been merged with another one?

Rainboe
12th Sep 2007, 16:53
Hopefully thread ended- I find the manner of querying it slightly offensive- it was more a burrowing exercise than an outright question. Where ever did 'rushed'/'messing up alignment' come into it?

Nicholas49
13th Sep 2007, 17:58
Rainboe:

I raised the question because I wanted to know the answer, not because I was "burrowing". What a ridiculous assumption. If you check the name on the forum, you'll see that it's designed for questions. If you don't want to answer questions, then don't.

Bullethead, Caudillo and chiglet:

Thank you very much guys for your helpful, informative explanations.

ChristiaanJ
13th Sep 2007, 20:17
On Concorde, you couldn't even do a "run-up on the brakes"... the aircraft would have literally started sliding forward.
So you lined up, did your last checks, released the brakes and slammed the trhottles forward, then very carefully monitored spool-up, reheat lighting correctly, etc., during the first few seconds of the take-off roll.

Rainboe
13th Sep 2007, 22:05
Nicholas49, I did take the trouble to answer your question, so i assume you were still thankful to me for doing so, even though I disliked your reference to
<<Is it at all a sympton of a rushed take-off?>>....an offensive comment you did not need to add. You could have simply asked the question without trying to burrow and discover what you thought may be a defective operation!

I have now removed my answer.

Nicholas49
14th Sep 2007, 15:53
Rainboe,

I was of course grateful for your taking the time to answer the question.

I asked whether the practice was the symptom of a rushed take-off because it can appear that way to passengers. I remember when aircraft always waited on the runway before taking-off.

To clarify: by "rushed" I did not mean to imply inept piloting. Instead, I thought this practice may have been the take-off equivalent to the intensive-braking used to vacate a runway quickly when aircraft are waiting to land.

It is clear from everyone's answers that the practice is standard.

It was certainly not intended to be offensive.

Rainboe
14th Sep 2007, 16:02
I understand. No aircraft need wait on the runway- pre-takeoff checks can be rapidly completed. The only reason there is a pause before take-off is to wait for a preceding aeroplane to take-off and get clear, or one to land and clear before brakes release. Powering up on brakes is not desired or recommended, though an intermediate power may be required in icing conditions to ensure the engine de-icing can work long enough to ensure there are no problems during high power for take-off. So the ideal is to turn onto the runway and keep it rolling while you spool up the engines, without touching the brakes.

clifftop
14th Sep 2007, 16:06
Having watched hundreds of aircraft take off from LPL whilst listening to comms from tower - pilot, It is a fairly regular thing to hear the pilot say "ready on reaching" as he approaches the holding point short of the rwy.
I have also heard the ATCO ask the pilot "are you ready for immediate departure?" this is often when there are aircraft on finals to get the departure off the ground sooner rather than later.

I am aware that LPL is under certain jurisdiction from Manch, there is usually a delay between each IFR departure as dictated by Manch.

Nicholas49
14th Sep 2007, 16:27
Thanks, Rainboe.

magpienja
14th Sep 2007, 17:57
Hi guys I was watching some landings a Liverpool this morning runway wet, a 737 and a airbus think it was a 320 landed while I was there, on touch down with both aircraft I could hear a screeching noise sounded like tyres skidding for quite a few seconds after touchdown,

Would that be what I could hear, if so do the brakes come on automatically.

Nick.

BOAC
14th Sep 2007, 18:14
Just the anguished sound of rubber accelerating from standstill to touchdown speed. 'Braking' in reverse if you like.

FlightDetent
15th Sep 2007, 09:24
ATCO ask the pilot "are you ready for immediate departure?" Perhaps it may be intresting to know that "immediate departure" is in fact an estabilished code phrase used to describe rolling take-off Rainobe and Nicholas have been discussing so eagerly.

clifftop
15th Sep 2007, 11:55
That's why I posted the reply. :rolleyes::rolleyes::rolleyes::ugh::ugh:

NW1
20th Sep 2007, 21:08
For supersonic acceleration (Mach 1 to about Mach 1.7), Concorde needed all four reheats (too much asymmetric thrust with three).
If one didn't light you could still do the accel. If two failed it was unlikely to work - what dictated whether it worked was not the number of reheats but getting to M1.7 by the 15' accel limit on reheat use - there was no problem with assymetrics. Actually somedays (e.g. cold OATs) the accel could outpace the fuel transfer, the technique then was to switch off one reheat to allow the fuel to catch up...
So if for whatever reason one of the reheats didn't light at the start of the take-off roll (which occasionally did happen), the take-off was abandoned, and the aircraft would return to the stand.
Again, not quite right. It was a matter of performance - if the TOW was a certain margin below PLTOW (using a graph in the performance manual) then you only needed 3 reheats (called a "GO" takeoff), otherwise you did need all four (called a "STOP" takeoff). This check was made at 100kts not 60kts - the NHP called "100kts" and if all was well the E/O called "power set" and off you went. Interestingly, a single reheat failure after 100kts did not constitute an engine failure and you carried on irrespective of TOW (rare: once lit they tended to stay lit - it was getting them lit that could be problematic....). Two reheats out constituted an engine failure and was treated in the conventional way...
Such a rejected take-off was a pretty straightforward affair, because the aircraft would usually not have reached more than about 50mph
See above - the reheat GO/NO-GO decision was made at 100kts.
Offered purely out of interest - hope it helps...

ChristiaanJ
20th Sep 2007, 21:37
NW1,
Many thanks!
And apologies to all for providing less than exact info.
CJ

NW1
21st Sep 2007, 12:29
...you're welcome - don't apologise; it's only a chat!

ChristiaanJ
21st Sep 2007, 14:39
...it's only a chat...Sure, but as long as people are still interested in the subject and chat about it, it's always nice to get the facts right :8 !

xxgunnerxx
5th Oct 2007, 02:55
Hi,
What are the possibilities of a quad engined airplane (744, 340s only) having a simultaneous or one after the other engine failure during takeoff and the speed is v1-10~20kts? Would the airplane be able to climb on 2 engines if its fully loaded? if so what would be the biggest rate of climb in feet per min? what is the biggest of climb on these airplanes if only one engine fails?

Another question kind of relating to the above is what if the PF is the FO and an emergency occurs while taking off, would the captain immediately say "i have control" and perform the takeoff, and do the rest of the emergency, while the FO just helps with ATC and checklists?

Bullethead
5th Oct 2007, 07:30
G'day xxgunnerxx,

Years ago when I first started on the B747-400 we finishd a sim session a little early and had a bit of time to play. The instructor set the a/c at MTOW and the temp 30 degC then during the takeoff run had an engine fail after V1 and the other one on the same side fail at VR. So a double asymmetric engine failure at max weight on a hot day.

It was a handful to say the least and if you followed the flight manual it resulted in a gradual descent into the ground, however if you selected the flap up from F20 to F5, regardless of the speed, then there was a huge reduction in drag and the a/c would eventually accelerate and climb away as you cleaned up further. Once you got it clean there was more than enough thrust to complete the approach.

Second paragraph, if the take off is rejected the captain takes over, however if the take off is continued and the FO is handling things well he continues to fly and set up for the approach, though in my company the captain has to do the actual landing.

Regards,
BH.

Rainboe
5th Oct 2007, 09:33
Gunner- you need to understand what V1 means. In the scenario you suggest, no chance, therefore you should already have abandoned your take-off. Particulary with both on one side out, you would not be able to maintain runway direction at that speed. As Bullethead stated, at V1 you have a severe struggle on your hands, but it can be done. In real life, a significant number, probably a majority, of such failures would result in a crash.

RJ Kanary
16th Oct 2007, 22:32
A less aviation inclined acquiantance of mine posed to me a question about something he observed on a flight home from LAS.He noticed the spoilers being deployed while in what appeared to be normal flight.What situations would exist for the spoilers to be deployed in flight?
Landing is the only time I've seen spoilers active, but then I can count on the fingers and thumb of one hand how many times I've flown. <G>
Thanks for your time.............. :)

Diaz
16th Oct 2007, 22:49
I'm no expert, but I think if the pilot/ATC made a (big) mistake and they had to make a steep descent or quick deceleration, they can be extended to increase drag, but you don't see it done very often.

4potflyer
16th Oct 2007, 23:12
I've seen them deployed in flight on a 737 IIRC.

If you are asked to get a move on by ATC before starting a descent, or if you are given a late descent, why not use them?

Ideally probably would not want to as its effectively 'wasted' energy, but if it gets the desired effect for the situation, I don't think anyone has made a 'mistake'.

Disclaimer: I've never flown anything with spoilers on.

Bealzebub
16th Oct 2007, 23:40
On a 757 the spoilers are used to increase drag and reduce lift in flight and on the ground. However they also supplement the ailerons in roll control depending on the amount of control wheel input. Therefore in a turn you will very like see them deploying without any direct input by the pilot other than through the normal control wheel input.

In a descent, or when slowing down in level flight, or when slowing down and descending they will be selected by manual input of the speedbrake lever. This is an almost every flight occurance at some point or other.

Finally the spoilers are fully deployed automatically ( normally ) or manually on landing to reduce lift and transfer weight to the main wheels (and brakes). This occurs as soon as the main gear trucks untilt, or when either thrust reverser is activated, or when manually applied.

It is not likely to be the case they were deployed my mistake, as to do so would be very difficult and extremely unlikely. They are used very often and normally to achieve a desired profile by either increasing a rate of descent, or increasing the drag to slow the aircraft down. The 757 is quite a slippery ship, and often needs assistance in reducing speed particularly in busy traffic environments or where absolute restrictions apply, such as in the USA (and other locations) where a mandatory 250 kts or below is a requirement below 10,000 feet for example.

RJ Kanary
17th Oct 2007, 01:24
Thanks for the informative replies. :)

Lax Pax
24th Oct 2007, 17:31
Why are (at least some) airliner engines mounted at an up angle to the horizon--the intake end higher than the nozzle end? I first noticed the design element on the MD-80.

http://www.simviation.com/pageimages/md80a.jpg

Apologies if the question is answered elsewhere on Pprune--I did look, and couldn't find it addressed.

Many thanks.

Lax Pax

Nathan Parker
24th Oct 2007, 17:51
Why are (at least some) airliner engines mounted at an up angle to the horizon

Generally aligned with the direction that the air is flowing around the fuselage.

Lax Pax
24th Oct 2007, 18:04
Thanks for the reply, N.

So then wing-mounted engines would not be angled because there's no fuselage and associated air stream to deal with(?).

Nathan Parker
24th Oct 2007, 19:30
So then wing-mounted engines would not be angled because there's no fuselage and associated air stream to deal with(?).

The airflow ahead of the wing is an upwash, whereas the flow behind the wing is a downwash. If the engines on the wing are angled at all, they should be pointed slightly down. I haven't noticed if this is true or not. Too much of an angle would cause the jet exhaust to impinge the underside of the wing.

Tootles the Taxi
24th Oct 2007, 20:22
Not meaning to hijack the thread but I've recently noticed an MD80(2/3?) with engines removed; there are 3 windows aft of the left main door (assume same on stbd side). Having not noticed them before I presume they are masked when engines mounted. What would be located inside the cabin here? Thinking a galley perhaps?

Thx.

barit1
24th Oct 2007, 21:55
Sometimes there's a galley or such there, but mre likely there are SLF seats behind the engine inlets.

If you want something of an education in fluid flow, sit back there during a night takeoff in light snow, and watch the snowflakes change direction as the ship accelerates down the runway. REALLY interesting!

AnthonyGA
17th Nov 2007, 00:43
What are the reasons for having different idle speeds for jet engines in different flight regimes? I note that there is a ground idle, which seems to be about 25% N1, and a flight idle, which seems to be around 40% N1, and both seem to be automatically set for idle in recent aircraft (I suppose pilots of older aircraft set these speeds manually?).

I've read of two potential reasons for the difference: (1) Higher idle speeds at high altitudes reduce the possibility of a flameout; and (2) flight idle is designed to match the drag of engine nacelles so that an aircraft can glide in a descent with the drag of the nacelles "erased."

Are these the real reasons? Are there others? Is the notion of flight idle and ground idle common to all jet aircraft, or only certain aircraft? Are there other idle speeds?

Intruder
17th Nov 2007, 02:26
Another reason is a safety buffer, since it takes a LONG time for a CF6 (for example) to accelerate from 25% N1 to 40%.

There is also a consideration in some airplanes of reliably providing enough bleed air for all the systems airborne.

SNS3Guppy
17th Nov 2007, 07:29
Anthony,

Some aircraft use different idle settings, others don't. Generally that's an engine function, rather than an aircraft function; airframes tend to use a variety of engines either as a customer choice or just as the airplane evolves...and what's an option on one engine installation may not be the same on another...even though it's the same type airplane. The B747, for example...three different basic engine makes to hang on it, with differnt models engine from different manufacturers.

I flew the C-130. The early models I flew didn't have a ground idle. These were turbopropeller airplanes; the T-56A-9 engine had no ground idle, but the T-56A-14 engine did. It was quieter, cooler, and burned less fuel on the ground. As previously mentioned, in flight, a higher idle speed means the engine spools up faster, and is less likely to flame-out...as well as producing more bleed air for pressurization, air conditioning, anti-ice, etc.

At low engine speeds, small openings have to be used in the engine, sometimes called acceleration bleeds, which dump air out of the compressor to keep it from stalling. As air enters the engine, it's compacted or compressed, and pushed through a series of stages which continue to do so, before it's slowed, pressure increased, and then fuel added to burn. In those compression stages, an orderly flow of air is necessary; increase the pressure too much without enough airflow, or allow the blades to turn slowly enough, and the airflow in there stalls; it can reverse direction in the engine, the flame in the engine can go out, and somewhere in there, engine damage can also occur. The acceleration bleeds are small valves which open to let some of that pressure out when the engine is operating at low speeds.

Wasting this compressed air in flight is pointless...especially when the bleed air is used for so many functions in the airframe. Preventing the engine from going so slowly as to open these bleeds is an important operational consideration, then, and is designed into the operating design of the engine. Some engines revert to ground idle on their own after touchdown, others must be put there manually.

AnthonyGA
23rd Nov 2007, 17:21
Is there some secret to getting a large aircraft to roll at a constant speed during taxi? I'm having a terrible time getting my aircraft to taxi at a reasonably constant speed, and I'm wondering if airline pilots have some special technique for doing this. Or is it just practice?

For example, it seems that a lightly loaded 747-400 will lazily start rolling at 35% N1, but then it will speed up and up and in no time you're rolling along at 30 knots. And yet if you set power to 32% N1, it will gradually slow down and stop. The "sweet spot" seems to be about 0.00008 knots in width. Sometimes I hit it entirely by accident, but otherwise constant tweaking of the throttles is required. I must be doing something wrong.

Ditto for the 737-800, except that you see results a bit quicker when you adjust the throttles. A 767-300ER seems to want to roll even at idle, but it does seem to speed up and slow down more rapidly, which makes it a little easier to set the throttle.

So, how do you taxi at a steady speed in real life? Do you try to find the right spot for the throttles or just constantly adjust them. And is it cheating to set the throttles so that you gradually accelerate and then tap the brakes a bit to slow down when necessary? I don't know how much this heats the brakes. On a small plane I'd use differential braking to help in turns but I don't know if this is allowed or a good idea on large aircraft (and in any case differential braking doesn't seem to make a lot of difference in turns, at least in simulation).

AnthonyGA
27th Nov 2007, 15:31
Is there some secret to getting a large aircraft to roll at a constant speed during taxi? I'm having a terrible time getting my aircraft to taxi at a reasonably constant speed, and I'm wondering if airline pilots have some special technique for doing this. Or is it just practice?

For example, it seems that a lightly loaded 747-400 will lazily start rolling at 35% N1, but then it will speed up and up and in no time you're rolling along at 30 knots. And yet if you set power to 32% N1, it will gradually slow down and stop. The "sweet spot" seems to be about 0.00008 knots in width. Sometimes I hit it entirely by accident, but otherwise constant tweaking of the throttles is required. I must be doing something wrong.

Ditto for the 737-800, except that you see results a bit quicker when you adjust the throttles. A 767-300ER seems to want to roll even at idle, but it does seem to speed up and slow down more rapidly, which makes it a little easier to set the throttle.

So, how do you taxi at a steady speed in real life? Do you try to find the right spot for the throttles or just constantly adjust them. And is it cheating to set the throttles so that you gradually accelerate and then tap the brakes a bit to slow down when necessary? I don't know how much this heats the brakes. On a small plane I'd use differential braking to help in turns but I don't know if this is allowed or a good idea on large aircraft (and in any case differential braking doesn't seem to make a lot of difference in turns, at least in simulation).

Rainboe
27th Nov 2007, 19:51
With light taxi power set, it takes too long to accelerate to a proper taxi speed. I use a fair amount of power to get speed up to a decent figure and control with brakes from then on. You do not use differential brakes on big jets, and even differential power is not usually recommended. During taxi there is no set power- speed depends on weight, wind direction and taxiway slope. You just have to take it as it comes and keep on top of it.

Intruder
27th Nov 2007, 20:15
You do not use differential brakes on big jets
Maybe you don't, but many of us do, especially on wet or slippery surfaces.

Avman
27th Nov 2007, 21:32
AnthonyGA, my experience is that FS does not accurately simulate aircraft taxi characteristics.

Intruder
27th Nov 2007, 23:27
PLEASE! If you're asking about simulators instead of real airplanes, make that clear in your post!

AtoBsafely
28th Nov 2007, 02:49
In the real world,

It takes a little extra thrust to get rolling, but once you are moving idle is pretty much all you need. On a flat you may need brake to keep speed under control, but the only time I use extra thrust is going uphill or around a sharp corner.

From mostly flat airports,

Joe

parabellum
28th Nov 2007, 03:37
The Boeing recommended way, for a light B757,767 and 747-400 is to get the aircraft moving and let it accelerate to no more than 20kts then firmly apply brakes and reduce to a fast walking pace, release the brakes and repeat. Am told it will minimise brake wear.

exeng
29th Nov 2007, 14:24
You do not use differential brakes on big jets Training on the B.777 for BA was to use one side to slow during taxi and the next time use the other side. (Back in the 1990's anyway) Something to do with wear on carbon brakes being less rather than helping in a turn.


Regards
Exeng

AnthonyGA
30th Nov 2007, 06:31
Thanks for the replies. Looks like I'm safe setting the throttle to produce gradual acceleration and then tapping the brakes to slow down as required, if that's the way Boeing says to do it. I was worried about wear on the brakes or heating them up too much, but apparently that's not an issue.

I've not tried differential braking much because I figure it isn't used often in real life on large aircraft, and replies here seem to confirm that. It doesn't appear to have much effect in the sim unless I press one brake considerably harder than the other (in a small Baron, the aircraft will swing to one side with even the slightest differential pressure). I'll have to try the alternating technique and see how that works.

I suspect the throttle quadrant I have for my sim is a bit stiffer than the real throttle levers, and the range in which the throttles must be adjusted to go from slowing down to speeding up seems to be very small. Perhaps it's easier to make such small adjustments with the real throttles.

I haven't tried differential thrust at all (it's too hard to move the throttle levers separately in the sim), so I'm glad to hear that it isn't frequently used, as I was wondering about that as well.

Of the several aircraft I fly in the sim (737-800, 747-400, and 767-300ER), only the 767 seems to want to roll spontaneously at idle. The other two will eventually coast to a very gradual stop at idle thrust, depending on the weight of the aircraft and other stuff, of course. I'll have to try them all empty sometime to see what that does.

xxgunnerxx
4th Dec 2007, 03:15
I'm wondering if the autoland capability that most large transport jets have is really needed. How often are airplanes approaching their final destination that have visibilities of under half a mile? (maybe 1/20 flights?) So all those other times the system is not being used and creates a loss for the operator... And for all the other times that the plane IS landing in poor visibility why can't HUDs be installed and pilot lands the plane using the HUD this is a much chepaer than have an autoland system.

Intruder
4th Dec 2007, 05:34
What makes you think a HUD is cheaper than an autoland system? What is the relative level of training and maintenance required for both? After all, the autopilot systems are already installed (2 or 3 for redundancy), and the integration into the autoland system is well understood. By contrast, HUDs are relatively new in the airline world, and certification is not a piece of cake...

HEATHROW DIRECTOR
4th Dec 2007, 07:18
In Low Visibility Procedures aircraft land "hands-off", ie fully automatically, thereby removing any requirement for an HUD...

Nick Thomas
4th Dec 2007, 11:08
Hi could someone please tell me why the opening side cockpit window is known as thr DV window? Thanks Nick

D120A
4th Dec 2007, 11:30
Direct Vision. What you see is what you want.

xxgunnerxx
4th Dec 2007, 14:04
i think its cheaper because mechanics and pilots have to constantly check if the EXTREMELY complex autoland system works, if it fails during low visibility - a diversion is needed, while in a HUD I'm quite sure that the system is much simpler since all it requires is to display primary display in another place (ie the HUD). I also don't think that the HUD system is new since a lot of old fighter jets already had that system and pilots coming from the military (back in those days) could have already known how to operate so the retraining would have been much cheaper for the operator, thus saving money once again.

HEATHROW DIRECTOR
5th Dec 2007, 14:41
So what would your proposed HUD display?? You still have to have the complex autoland equipment to get the aircraft on the ground.

With great respect, I don't think you fully appreciate the situation.. An HUD by itself cannot get the aircraft down in poor visibility - it's simply a way of displaying information to the pilot; a great deal of other equipment is required so an HUD cannot replace an autoland system.

xxgunnerxx
5th Dec 2007, 15:40
but when the pilot is using the the HUD he is landing the airplane himself..... so there is no need to have an autoland system...

FlightDetent
6th Dec 2007, 04:32
but when the pilot is using the the HUD he is landing the airplane himself..... so there is no need to have an autoland system...
Yes there would be. HUD merely enables the pilot to fly head-up and see instrument information while looking through the window at the same time. It doesn't solve the problem, of actually seeing what is in front of you. There is a formalised requirement that the minimium visibility for pilot to obtain sufficient visual reference of outside world so that s/he can correctly judge the aircraft position, velocity and trajecotry in order to land safely is 300 m. With a HUD and 200 m RVR, there still woul be no benefit. Autoland can solve this. Actually, have solved this 40 years ago.

FlightDetent
6th Dec 2007, 06:04
I'm wondering if the autoland capability that most large transport jets have is really needed. How often are airplanes approaching their final destination that have visibilities of under half a mile? (maybe 1/20 flights?) So all those other times the system is not being used and creates a loss for the operator

I would say 1/500 flights. But the operators decide what is best, to divert, not to fly at all, or pay for an autoland. Imagine you divert, crew runs of of duty time and in 2 hours the weather clears up and you have customers waiting for the return flight. The costs are immense and so is the competitive advantages for those with autolands. That's why manufacturers developed and successfully sell autoland equipment. These days it a well proven, easily obtainable, and operationally very benefical technology.

AnthonyGA
7th Dec 2007, 15:59
Talk of HUDs makes me thing of a question that has puzzled me for a long time: How do you keep the HUD aligned with the scene outside? I mean, if you are looking directly through the HUD, and the lines on the HUD match the contours of the runway ahead exactly, what happens if you move your head to one side? Don't the lines on the HUD shift out of alignment then?

Seems to me that either you'd need something to clamp your head in place relative to the combiner of the HUD, or the HUD would need some sort of optical system that somehow keeps the lines on the HUD in the right place no matter what the position or angle of your view through the combiner. So how is it done?

I'm not actually that keen on the idea of HUDs and I'm glad that they are not widely used.

Knife-Edge
7th Dec 2007, 23:22
I have been following the very interesting Iberia IB6166 BOS-MAD thread (http://www.pprune.org/forums/showthread.php?t=302971)and it has prompted a SLF de-icing question.

A couple of winters back I was on a TOM 737 flight from BOH to CDG. It was middle of the day no frost and about +3c. While we were boarding it started raining. After boarding the capt announced there would be a 'short delay for de-icing', which was a bit of a surprise. The Ryanair next to us departed without de-icing.

As the sector from BOH-CDG is short it may be that the a/c didn't require fuel (at ambient) and as TRT was short from the previous arrival would the decision to de-ice in positive ambient temperatures and rain be based on the potential below 0c fuel temperature in the wing from the previous flight causing freezing on contact? If so how does the PIC make this judgement call as I assume it takes a long time for fuel temps to normalise to ambient field level temps that are only a few degrees above freezing?

Itswindyout
8th Dec 2007, 14:13
the idea is that you use the eye liner, the three balls on the centre windscreen post, to ensure your head is in the correct spot.
BUT the HUD has a very small angle of view, so very often a small head movement moves the huds display out of the direct and correct line of sight.
Not normally a major problem, I find the worst problem, is the very slight lack of infinity, and thus a slight re focus is required. Now thats a problem.

BOAC
8th Dec 2007, 21:35
Knife-Edge - the decision on whether to de-ice is a complicated one. If your a/c had a longish inbound sector the wings would have been well below zero and any rain would freeze on them (known as 'rain ice'). Often an upload of 'warm' fuel (normally well above ambient temperature since it is normally stored underground) will clear the ice. The Ryanair may have had a short inbound sector or uplifted fuel - or both. One hopes that consideration was given to de-icing and I'm sure it was. Normally a combination of experience, indicated wing fuel temperature and a look at the way the rain behaves as it lies on the wing will make the decision. No doubt your Tom Captain made the correct decision:ok:

Yellow Sun
12th Dec 2007, 18:15
The except from the A320 FCOM answers your question:

FWC generates synthetic voice for radio height announcement below 2500 feet. These announcements come through the cockpit loudspeakers even if the speakers are turned off.




Predetermined call out

The altitude call out uses the following predetermined threshold :



height (ft) call out
2500 TWO THOUSAND FIVE HUNDRED OR TWENTY FIVE HUNDRED
2000 TWO THOUSAND
1000 ONE THOUSAND
500 FIVE HUNDRED
400 FOUR HUNDRED
300 THREE HUNDRED
200 TWO HUNDRED
100 ONE HUNDRED
50 FIFTY
40 FORTY
30 THIRTY
20 TWENTY
10 TEN
5 FIVE
DH + 100 HUNDRED ABOVE
DH MINIMUM

Pin programmings allow the operator to select the call outs needed.

If aircraft remains at a height that is in the detection zone for a height callout, the corresponding message is repeated at regular intervals.




Intermediate call out

If time between two consecutive predetermined call outs exceeds a certain threshold, the present height is repeated at regular intervals.

The threshold is : 11 seconds above 50 feet
4 seconds below 50 feet

The repeating interval is 4 seconds.




RETARD announcement

The loudspeaker announces RETARD at 20 feet or at 10 feet if autothrust is active and one autopilot is in LAND mode.



YS

AnthonyGA
22nd Dec 2007, 20:44
This is for anyone who has been to TNCM in a heavy, particularly a 747:

What are the real-world taxi procedures for TNCM for heavies? I've been asking around and looking around and I'm still not clear where the heavies turn around. Apparently they all go to taxiway Charlie, then backtrack along the runway to get to the start of runway 9, but I don't know if they take that little taxiway near the 09 end for their 360 to line up, or whether they backtrack all the way to the end of the runway and then turn around in the tiny space provided on the north side. Which procedure is used in real life? Or is the real-world procedure something entirely different?

Likewise, for landing, what's the usual procedure for large aircraft? Do they go all the way to the end of 09 and then backtrack to Charlie, or do they stop at that intermediate point where there's a space to turn around, or what? I have quite a bit of difficulty turning a 747 on either spot; with the tiller hard over and lots of brakes on the right side I can just barely squeak by, although the brakes are usually excessively hot on that side by the time I reach the gate.

anengineer
29th Dec 2007, 01:32
Whilst waiting on the spectators balcony at LPMA recently, I watched an Easyjet 737 arrive. As the engines spun down I could hear this god awful clunk with each revolution of the starboard engine. The ground crew didn't seem concerned, neither did the captain when he did a walkaround and gave the fan a spin with his hand (and the clunk was there as he did it).
I've never heard this before.
Any ideas on what it might be, and why nobody seemed concerned ?

FLCH
29th Dec 2007, 04:47
I think it's the fan blades making that noise, the blades are seated in the hub but have a small gap in them (presumably for expansion) and you hear a clicking or ticking noise when they move downwards. It is very prominent when a breeze is going through the engine. I will be the first to admit I may be wrong, maybe someone can correct or elaborate for me.

ISO100
11th Jan 2008, 10:45
There is a thread in the flightdeck forum discussing a recent incident at Changi.

It is suggested that pliots can not use braking if the AC is rolling backwards as the result could possibly be an unplanned rotation. :)

Is it not possible to apply brakes to the nose gear only to prevent any danger of the aircraft ending up sitting on its tail?

Bullethead
11th Jan 2008, 10:53
The vast majority of aircraft only have brakes on the mainwheels. Good idea though.

Regards,
BH.

treadigraph
11th Jan 2008, 12:22
I was once sitting in the back seat of a C172 parked facing up a bit of a slope. For some reason mein Kapitan released the handbrake before start up and we started to roll backwards towards the clubhouse. He applied the brakes quite heavily and, no doubt assisted by 200lbs of yours truly perched aft of the centre of gravity, we came awfully close to ploughing a narrow furrow with the bottom of the rudder! Not an auspicious start to a jolly...:rolleyes:

The C-17 demo at Farnborough one year included a reverse thrust exit from the runway followed by some enthusiastic braking and a distinct lifting of the nose gear! The crowd loved it.:ok:

dgutte
6th Feb 2008, 08:47
Flew out of Orly on Monday night and as we were pushing back I noticed that an AF 321 was being pushed back with the tug attached to the underwing left side undercarriage. I have never seen this before and it looked like an inefficient way of doing it.

Any ideas?

crisso
19th Feb 2008, 16:10
I recently flew to/from LGW to Innsbruck (LOWI) for a ski-ing holiday with Easyjet on an A319. Both ways the Flight deck crew announced themselves as comprising three members. Normally, of course, there would only be two. Any particular reason? I thought it may be due to being a new-ish route for Easyjet but, moreover combined with the 'interesting' approach to LOWI thus, perhaps they were familiarising pilots to same. Any insider/official explanations?

BOAC
19th Feb 2008, 16:27
You are correct. It is a Category C - ie 'more difficult' airfield - and special simulator training is required. A visit is useful as well and may be a requirement in EasyJet.

maffie
20th Feb 2008, 14:52
Hello,
Had some ITVV DVD's for Christmas and a question has just popped into my head (Some 8 weeks later)

When the pilots are checking the control surfaces during taxi, why don't they check the flaps and spoilers ??

ie. Why not FULL flaps and then return them to TO configuration.

Cheers :ok:
Matt

PPC
5th Mar 2008, 10:52
I was on a JET2 flight landing at LBA on Sunday (2 March) and the landing was quite bumpy due to wind conditions. Upon touch down we appeared to bank heavily to the right (felt like on one wheel!) and then levelled off. Everyone on board breathed a sigh of relief!

My question is how difficult is it to land in strong winds and are we in any danger from tipping over! As a passenger I feel quite nervous can anyone put my mind at rest?

Many thanks..

the dean
5th Mar 2008, 14:17
hi PPC,

been a flying instructor many years. we teach for these sort of things but sometimes a stronger than average crosswind gust may just strike when an aircraft is at its most vulnerable...that is just as it flares to land with consequential results but not always fatal nor damaging to aircraft nor passengers.

there have been in both light aircraft and large passenger jets crosswind induced accidents....but your chances of being involved in a car accident are still higher....

so relax...and enjoy your flights..when you a$$ is strapped to metal its going to do what its going to do in any event..!!:}

safe flying..

the dean.

Pilot Pete
5th Mar 2008, 14:47
upon touch down we appeared to bank heavily to the right (felt like on one wheel!) and then levelled off. It could have been that the pilot way just 'de-crabbing' the aircraft to align with the runway (using the rudder) and then putting in aileron control to put the into wind wing 'down' to maintain the centreline without drift. This is a recognised technique for the 737 and the 757 (which Jet2 have) and it means that you touchdown on one maingear before the other with the wing down. Nothing to worry about!

PP

p.s. should have been landing on Friday night, now that WAS windy!:ok:

easy1
10th Mar 2008, 22:20
It's very simple.
If the cabin is not secure then we DO NOT land. There is no way around it!!

justawanab
17th Sep 2008, 11:44
We all (presumably) know about stick shakers and audible stall warnings that tell when a stall has or is about to occur but what I am confused about is how a stall or potential stall is actually detected.

Is the actual physical stall somehow detected by sensors reading air pressures etc, or does some particularly complicated bit of computer wizardry determine that the aircraft should be stalling "about now" given it's airspeed, configuration and external climatic conditions?

Yay!!!
17th Sep 2008, 22:49
It is the air sensor on to top of the wing and senses the turbulent air foorewarning a stall

SNS3Guppy
17th Sep 2008, 23:34
We all (presumably) know about stick shakers and audible stall warnings that tell when a stall has or is about to occur but what I am confused about is how a stall or potential stall is actually detected.

Is the actual physical stall somehow detected by sensors reading air pressures etc, or does some particularly complicated bit of computer wizardry determine that the aircraft should be stalling "about now" given it's airspeed, configuration and external climatic conditions?


There's no sensor on top of the wing of which I'm aware, but methods of actually detecting a stall are nearly as varied as there are airplanes...and rightfully so, because airflow around each type of airplane is different.

A stall is an increase in angle of attack to the point of airflow separation, and a big buildup of induced drag and a loss of lift. These things don't happen all at once, necessarily. The key to understanding them is monitoring the angle of attack, and stall warning systems using stick shakers, pushers, etc, do this by sampling the angle of attack. They also sample the position of flight controls such as flaps...because the angle of attack at which the wing stalls will change with the flaps setting as the wing shape and configuration changes. It also takes into account air data information received from an air data computer. How the airplane does this depends on the airplane and the systems in use. Some are very simple, some are not.

Some airplanes like the airbus simply won't let the airplane stall due to "laws" built into the system. Some airplanes such as a single engine Cessna may use a small tab or even a simple suction port on the leading edge of the wing to produce a noise, and nothing more...and yet other airplanes don't even do that...they're simply detectable by a buffeting in the airframe or controls.

An angle of attack vane may be a movable vane which acts like a weathervane of sorts, or it may simply be a probe with holes that sample the way air flows around it to derive angle information. This is then applied to the aircraft data to determine what the actual angle of attack is (local angle of attack isn't the same as the free airstream, and AoA changes with maneuvering or configuration changes may mean that the AoA isn't the same as what the probe or vane is seeing at any given time...computers correct this). The system in use then takes this information in, and outputs whatever signal is necessary...such as a stick shaker or pusher signal.

When the pilots are checking the control surfaces during taxi, why don't they check the flaps and spoilers ??

ie. Why not FULL flaps and then return them to TO configuration.


What's checked really depends on the airplane. The airplane will fly with or without flaps...but not with or without ailerons, rudder, and elevator.

We check all the flight controls after we've set the flaps to their takeoff setting. In cycling the flaps we also move a number of other parts, including the leading edge devices which cycle in stages. We can only test certain things with flaps up, some with flaps out...and we have specific set procedures for everything. In some airplanes, spoilers or aileron augmentation is tested at the same time...happens on our airplane. On others we test them and then test emergency stow systems...some thrust reversers are tested and stowed, and on others, they're not.

The primary reason for testing the flight controls is to make sure they're powered, and that they're not locked for any reason. For example, in our airplane, when the flaps are up, the outboard ailerons are locked out. We have to extend the flaps to get the ailerons to work. We're checking for freedom of flight control movement and that our indicators in the cockpit correspond. There's no reason to run the flaps down, especially during ground operations when there's a greater possibility of damage, hitting something, etc. More importantly, we certainly don't want any possibility that they're overextended and a takeoff roll started with them in that dangerous condition. Further, there's no guarantee that simply because they extended before flight, that they'll work when it comes time to land.

Especially in a big airplane, moving flaps is a big deal, especially on the ground. Even powering a system, such as electrical or hydraulics, can have severe consequences...or moving a flight control for that matter. What's moved, when, where, and how fast must be carefully considered at all times...especially on the ground. We have flight control surfaces longer than a bus. Our horizontal stabilizer and elevators are bigger than the wings on a DC-3, with a wider span. There's a lot of metal moving out there. The flaps extend down a considerable distance, and there's just no good reason to put them down and then up again.

As the engines spun down I could hear this god awful clunk with each revolution of the starboard engine. The ground crew didn't seem concerned, neither did the captain when he did a walkaround and gave the fan a spin with his hand (and the clunk was there as he did it).


Fan blades generally sit loose in the hub, and tend to rattle when they're not under a load. This is done for several reasons, including issues regarding thermal expansion. hearing a rattle or clicking sound isn't at all unusual, when the engine winds down or is windmilling on the ground. You may hear other sounds, depending on the type of engine, associated with other parts of the engine, or even accessories.

Rainboe
18th Sep 2008, 07:39
Guppy, you'll make a fortune if you write a book all about aviation! You explain it so well! Even I learn!

VnV2178B
18th Sep 2008, 09:19
Rainboe, Guppy,

I will second that comment - it's so refreshing to have a proper expert on the forum and one who doesn't patronise or critcise even the simplest of questions, that book sounds like a great idea!

thanks for all your input:

VnV...

SNS3Guppy
18th Sep 2008, 09:33
Geez...you guys are making me turn red. Thanks! :uhoh:

VAFFPAX
18th Sep 2008, 10:38
And humble too. Bless.

Seriously though SNSGuppy, your explanations are a shining example to others on how you can bring your point across without being rude or snobbish about it.

I love reading your posts. :-)

S.

AnthonyGA
21st Sep 2008, 04:44
What's the secret to finding your gate when you land? Do you have special charts that give the exact location of each gate, or does someone discreetly show you the way, or what? I've seen airport diagrams but they just show runways and taxiways and things like that, not individual gates. I don't recall ever seeing a "Follow me" car or anything in front of the aircraft on actual flights I've taken. And it seems unlikely that pilots have memorized the position of every gate at every airport. So how do you find your way without getting lost or having to turn around or anything like that?

ManofMan
21st Sep 2008, 09:19
Anthony,

When the aircraft is switched to ground frequency the controller will advise them what stand they are parking on and how to get there...ie "Take taxiway alpha, then juliet parking stand 214", as the aircraft gets to the end of the taxiway (or near it) then the gates are clearly signed.

Just a spotter
22nd Sep 2008, 19:35
Hi

just wondering under what circumstances would an aircraft identify itself in radio communications using its airline ICAO code instead of it's airline Callsign, e.g. "Yankee Yankee Yankee 123" instead of "WingandaprAir123".

JAS

FlightDetent
23rd Sep 2008, 08:06
There is an ICAO list of assigned operator callsigns. If your operator is on it, the callsign is used (not that it is always the case).

Icao doc 8585 http://dcaa.slv.dk:8000/icaodocs/Doc%208585-143/Part%202%20-%20Telephony%20Designators%20-%20Decode.pdf (3,2mb)

FD (the un-real)

Just a spotter
23rd Sep 2008, 10:15
Thanks for the reply and link FD, but it doesn't quite answer my question (althogh, strangely enough, the airline in question isn't listed in the document you linked, but I can get it's ICAO info through google).

Assume an airline; "Flaky Air", with ICAO code YYY (yes I know that's reserved) and callsign "WingandaprAir". My (all be it limited) understanding is that on all flights operated by the airline (with the possible execption being wet-leases to another operator) in all radio communications with ATC the aircraft would identify itself as "WingandaprAir <Flight Designator>". So, under what circumstances would they delibertily identify the flight as "YYY <Flight Designator>"?

The only reason I asks is I overheard a flight using its ICAO code rather than callsign in all ATC communications and I'm fairly sure it was intentional, as at one stage the pilot communicating with ATC initially used the airlines callsign, then restarted the messages using the ICAO code. Before anyone gets uppity, it's not a conspiracy, I'm not questioning the abilities or motives of those involved. Its nothing more the idle curiosity (and my choice of hypotetical airline name and callsign is not intended to infer any view on the airline or individuals ivolved, it is intended to be light hearted).

JAS

SNS3Guppy
25th Sep 2008, 19:09
I don't recall ever seeing a "Follow me" car or anything in front of the aircraft on actual flights I've taken. And it seems unlikely that pilots have memorized the position of every gate at every airport. So how do you find your way without getting lost or having to turn around or anything like that?


Believe it or not, airplanes do get lost...turn the wrong way, take the wrong taxiway. One of the worst air disasters in history involved two large airplanes occupying the runway at the same time, when they ought not have.

For finding the gate, we call in advance, and have that information given us over the radio, a satphone or a datalink. Aeronautical charts publish gate positions, including diagrams and coordinates. Part of the airport area arrival is reviewing the area charts, arrival procedures, approaches, and then after landing, how we exit the runway and how we get where we need to go. Navigation on the ground is taken very seriously, because even at less than 20 knots, one can still get into a lot of trouble my making a wrong turn. We brief the taxi route, potential trouble spots with intersecting taxiways and runways, and then both pilots have an airport diagram available, with one looking outside, and one dividing his attention inside and out, and confirming each taxi turn and crossing.

We do get marshalling cars frequently, depending on where we go. They seem more common in Europe than most places. We also have airport signs marking taxiways and runways, and painted markings identifying gates and taxiways.

BelArgUSA
25th Sep 2008, 19:27
You get a "Follow Me" car coming in or going out in Russia...
Same for the CIS nations, and in China.
Not been to India recently, but was a "Follow Me" in Bombay for the cargo ramp.
Nice to have. I got lost a couple of times on unfamiliar airports.
Even once while taxiing a 747, at my "home base". Shame on me.
Of course, you young geeks and nerds never make mistakes.
:(
Happy contrails

FlightDetent
26th Sep 2008, 09:04
So, under what circumstances would they delibertily identify the flight as "YYY <Flight Designator>"?

Sometimes the ATC is unfamiliar with the callsign and may respond to our call using the letters. Once they do I would stick to it to make thing easier. It is the content of the message which is important, not the label. Otherwise, I have no clue.

PaperTiger
26th Sep 2008, 15:19
I don't know if Volga-Dnepr have a "call-sign", but their flights we get out here always communicate as Victor Delta Alpha nnnn.

Just a spotter
26th Sep 2008, 20:46
FD & PT

Thanks!

JAS

AnthonyGA
27th Sep 2008, 20:09
How do you decide how much spoiler to deploy, and when? Do you adjust the spoiler setting between the stops (between ARM and FLT, for example) or do you always advance it to the flight position if you need it at all? I know that there are guidelines for flap schedules but I haven't seen anything about spoiler schedules, so I assume pilots use them entirely at their discretion (?).

Also, in my sim I use spoilers mainly to slow down or to improve my rate of descent. Are there other clever uses of spoilers that I haven't discovered?

SNS3Guppy
27th Sep 2008, 22:10
Anthony,

That really depends on the airplane and the application. Spoilers serve a number of functions, including acting as speed brakes, devices to dump lift and put weight on wheels for landing, and serving to aid the ailerons in lateral or roll control in flight.

The way the spoilers are used varies with the airplane. Some airplanes use a set amount in flight, others use variable amounts in a speed brake capacity...some use them for roll control, others don't.

In our airplane, the speed brakes are auto spoilers on takeoff, and they're not armed. If we reject the takeoff, the spoilers will autodeploy when the second and fourth reverse levers are moved past the interlocks and into reverse. On landing, however, we arm the autospoilers and they deploy when the first and third thrust levers reach idle, and the weight is on the wheels enough to untilt them.

In flight, we use spoilers during a descent when necessary. It's generally preferred to not use them unless necessary; they make noise and vibration, and it's better to plan the descent and arrival without them. When given a late descent or a reduction in speed, however, they're a legitimate and useful tool for slowing down or going down, or both.

Lookforshooter
10th Oct 2008, 04:40
Some thoughts on why poeple leave thier gear down a little longer...they forgot...they are trying to let the water/snow ect spin off before they put the gear up in the well...cooling..they have a problem with an indicator.....but anyway you cut it...gear up for second segment is the rule...so leaving it down, messes with the numbers....

SNS3Guppy
10th Oct 2008, 23:32
If one plans for a gear down departure or delay with the gear and uses performance calculation accordingly, how does this "mess with the numbers," oh mighty technical one?

If one meets the climb gradient criteria, is this particularly relevant, and if one exceeds the certification second segment criteria during a diverse criteria departure, is it particularly important?

Lookforshooter
11th Oct 2008, 02:34
Oh really?...you got some approved 'gear down second segment climb numbers' ' oh knowlegable one?

SNS3Guppy
11th Oct 2008, 05:14
Yes, we do. It's all calculated using our Onboard Performance System...and we account for everything, including gear extention.

Of course, if the climb performance is in excess of the climb gradient required, then it's really irrelevant.

Whereas second segment performance begins with gear retraction and extends through typically 400-1,500' (depending on obstacles), if one hasn't retracted the gear and has first segment performance data, then one may use that. If one's performance is far in excess of the required gradient, and if one is using diverse criteria (you do know what that is, right??), then it's largely irrelevant.

You're so technical. "Messing with the numbers." Imagine what you'd say if you really knew what you were talking about!

Lookforshooter
11th Oct 2008, 20:36
Onboard Performance System...would that be like in your FMS? and you can just select gear down for second segement to give you a new V2 speed that you load into the tapes, and actualy get a new climb gradiant that you check againt the SID? IF so...thats great, I would like to know which FMS your using..having used Collins, Honeywell, NZ2000s...I would very much like you show me how to pull up that info. Thanks.
Given that I have never heard of second segement numbers being calculated for gear down until clear of obsticles, I would really like to learn more about this.

SNS3Guppy
11th Oct 2008, 21:00
Onboard Performance System...would that be like in your FMS?


No, it wouldn't. Not in the FMS, nor "like in the FMS."

Given that I have never heard of second segement numbers being calculated for gear down until clear of obsticles, I would really like to learn more about this.


Too bad. You probably never will. You don't have this capability on your microsoft flight simulator?

Lookforshooter
12th Oct 2008, 03:09
Oh, well...since it's not in the FMS, and not in my Part 25 manual...I am still wondering where those second segment gear down climb gradient numbers are?

SNS3Guppy
13th Oct 2008, 03:00
Whereas lookforshooter has been banned, look for nonflushinglav now...it's his new name. Same poster, same tired rhetoric.

For the record, there's no Part 25 manual. A transport category airplane is certificated under Part 25...but it's not a part 25 manual, nor does it make implication regarding what other aircraft types, or Part 25 for that matter, utilizes for performance data.

Then again, one can't expect to find all the relevant data when one is attempting to operate a microsoft flight simulator.

AnthonyGA
13th Oct 2008, 21:16
What is an Onboard Performance System, and what type of aircraft is it in?

SNS3Guppy
13th Oct 2008, 22:12
OPS isn't "in" an aircraft. It's a performance calculation system developed by Texetron, adapatable to various aircraft, and using manufacturer data.

With this system in place of spagetti charts, we can calculate our performance rapidly based on a lot of factors. What OPS does is allow us to use one source and combine every chart and piece of data into one calculation to come up with takeoff and landing data, as well as calculate max takeoff weights based on anticipated fuel burn, etc. It allows input of forecasts and reports, as well as inclusion of Configuration Deviation List items not applicable to a MEL...small things which of their own accord may not mean much, but which can add up to aerodynamic and performance penalties.

This includes gear extended, wet runways, close-in obstacles, etc.

What's important to understand is that OPS doesn't introduce something that wasn't there to begin with...in other words, anything we can do with OPS, we could have done with the manual charts, but in a much shorter time period, with a much higher degree of accuracy and repeatability. It even formats and prints out the data cards. It's faster than tabulated charts (tab data), whiz wheels, slide rules, etc. It's particularly helpful when a runway change, or even an ATIS change, alters the data used in the last calculation.

What I've often done in the past when I haven't had a tool like this to use, has been to carry my own book of the relevant performance charts, arranged in the proper order to allow me to rapidly make performance calculations on the fly; I've done this for each airplane I've flown, big and small.

Some airplanes provide less data than others. However, if one knows that one's performance is far in excess of that required, then minor variances in the predicted gradient are insignificant. Part 25 establishes minimum gradients. Terminal procedures establish minimum gradients and climb criteria based on obstacles, noise abatement, and air traffic control requirements. An aircraft which exceeds these minimum values by a significant number does not encroach on or compromise the minimum numbers when encountering a small degredation in climb performance.

Some manufacturers publish data with wheels down, some don't. A first segment climb ends with gear retraction, however, and where first segment data is published it is relevant.

So far as gear down operations go...it may be left down for many reasons, and a number of manufacturers do include gear down data. It may be a separate chart, or it may be a performance note showing the performance penalty to add for gear down. OPS simply makes it easier and faster to calculate.

FlightDetent
14th Oct 2008, 06:40
SNS3Guppy: I really want that book you would write. As my PPrUNe habits change, I realize that I skip threads and only stop at your posts to read. And then find there is nothing to contribute; and learn. :D

SNS3Guppy
14th Oct 2008, 08:04
Flightdetent, that's very kind of you to say, but it would become a very one-sided (read boring) conversation if just one of us posted all the time, wouldn't it?. What makes a site such as this great is the diversity of experience, background, and opinion.